Find the Mean and Variance of Random Variable Z = (5x+3)

Click For Summary
To find the mean and variance of the random variable Z = (5x + 3), the discussion emphasizes the need to compute E(X) first, which is essential for calculating Var(X). Participants suggest using theorems and corollaries related to expected value and variance, indicating that these formulas are crucial for the solution. There is a focus on ensuring that the values for E(X) and Var(X) are known or computed before proceeding. The conversation highlights the importance of understanding the underlying statistical concepts for accurate calculations. Overall, the thread centers on clarifying the steps needed to derive the mean and variance using the provided data set.
rogo0034
Messages
36
Reaction score
0

Homework Statement


Find the Mean and Variance of Random Variable Z = (5x+3)

Using data set:
AwPGYl.jpg
Using:
eRRWt.png

&
8gkTw.png

The Attempt at a Solution

 
Last edited:
Physics news on Phys.org
Sorry the data set it so large, I've been trying to adjust it to no avail.EDIT: Fixed it, should be easier to view now
 
Last edited:
rogo0034 said:

Homework Statement


Find the Mean and Variance of Random Variable Z = (5x+3)

Using data set:
AwPGYl.jpg



Using:
eRRWt.png

&
8gkTw.png




The Attempt at a Solution


You have a very small table of x and f(x) values. What is stopping you from computing EX? Why don't you just compute EX, then use that value in the computation of Var(X) = sum f(x)*(x-EX)^2 ?

RGV
 
They want us to use the theorem and corollary, any ideas? I'm not too savvy with this stuff yet.
 
rogo0034 said:
They want us to use the theorem and corollary, any ideas? I'm not too savvy with this stuff yet.

OK, so use the theorem and the corollary. Does the first theorem have E(X) in it? Do you know the value of E(X)? If not, you need to compute it. Does the second theorem have \text{Var}(X) = \sigma_x^2 in it? Do you know the value of Var(X)? If not, you need to compute it.

RGV
 
Question: A clock's minute hand has length 4 and its hour hand has length 3. What is the distance between the tips at the moment when it is increasing most rapidly?(Putnam Exam Question) Answer: Making assumption that both the hands moves at constant angular velocities, the answer is ## \sqrt{7} .## But don't you think this assumption is somewhat doubtful and wrong?

Similar threads

  • · Replies 2 ·
Replies
2
Views
1K
Replies
7
Views
2K
Replies
0
Views
863
  • · Replies 8 ·
Replies
8
Views
2K
  • · Replies 1 ·
Replies
1
Views
1K
  • · Replies 1 ·
Replies
1
Views
2K
Replies
1
Views
1K
Replies
9
Views
4K
  • · Replies 2 ·
Replies
2
Views
2K
Replies
2
Views
2K